Tndlive Medicine1

Download as pdf or txt
Download as pdf or txt
You are on page 1of 45

QuesID: -1

A 70-year-old man with Hypertension wakes up with severe chest pain and
diaphoresis. On examination he has bounding pulses with wide pulse
pressure. A diastolic murmur is heard along the right sternal border. Which
of the following is the possible etiology?
1. Aortic dissection
2. STEMI with papillary muscle dysfunction
3. Myocarditis with functional regurgitation
4. Flash pulmonary edema

QuesID: -2
A patient has got a bioprosthetic mitral heart valve. What will happen to his
heart sounds?
1. Loud S1 and normal S2
2. Soft S1 and normal S2
3. Absent S1 and normal S2
4. Normal S1 and normal S2

QuesID: -3
What is electrical storm?
1. > 3 episodes of ventricular tachycardia within 24 hours
2. > 3 episodes of atrial tachycardia within 24 hours
3. Electromechanical dissociation
4. Electrical alternans
QuesID: -4
Which of the following is effective in management of patients of Heart
failure with preserved ejection fraction?
1. Digoxin
2. ARB with neprilysin inhibitor
3. Isosorbide mononitrate
4. Phosphodiesterase inhibitor

QuesID: -5
Which of the following is incorrect about RCM?
1. Increased atrial size
2. Normal LV wall thickness
3. Reduced ejection fraction
4. Increased LV diastolic dimension

QuesID: -6
A 30-year-old male presents to ER with multiple episodes of chest pain,
lasting between 10-20 minutes over the last 2 hours. He admits that he did
“a bit of coke” at a house party. Clinical assessment reveals:
Pulse: 95/min, Blood pressure: 135/93mm Hg, sP02= 97% on room air.
ECG during a further chest pain episode reveals ST elevation in leads V2-
V6 and Troponin T is negative.
Which of the following is the most appropriate intervention for this patient?
1. Percutaneous coronary intervention (PCI)
2. Enoxaparin
3. Calcium channel blocker
4. Atenolol
QuesID: -7
Which of these statements regarding bicuspid aortic valves (BAV) is
FALSE?
1. BAVs are one of the most common congenital cardiac abnormalities
with a prevalence in the general population of up to 2%
2. Patients with BAV often need aortic valve intervention at an early age
than those with tricuspid aortic valves
3. BAVs are associated with aortic dilatation and aneurysm formation
but there is no increased risk of rupture or dissection
4. They are commonly associated with coarctation of the aorta (up to
50%)

QuesID: -8
A 75-year-old man presents to his GP for a routine check-up. On
examination the GP hears a high-pitched diastolic murmur, heard loudest
at the right sternal edge and on expiration. Given the likely diagnosis, which
of the following is NOT a risk factor for this condition?
1. Ankylosing spondylitis
2. Marfan’s syndrome
3. Holt Oram Syndrome
4. Infective endocarditis
QuesID: -9
Comment on the diagnosis of the CT chest shown.

1. Massive pulmonary embolism


2. Submassive Pulmonary embolism
3. Aortic dissection
4. Left atrial Myxoma

QuesID: -10
A 50-year male presented with high BP of 160/100 mm Hg and heart rate
of 120/min. CECT is shown below. Which is the best management of this
condition?

1. Surgical repair
2. LMW heparin
3. Beta blocker
4. Vitamin K inhibitors
QuesID: -11
An 80-year-old person has developed a PCA territory stroke. On
Ambulatory Holter he is found to be having atrial fibrillation. What
calculation should be used to evaluate for renal insufficiency of the patient?
1. NIHS score
2. ABCD2 score
3. CHADS2- VaSc score
4. Cockroft Gault formula

QuesID: -12
Straight back syndrome is associated with?
1. Mitral valve Prolapse
2. Ankylosing spondylitis
3. Osteopetrosis
4. Cor-triatriatum

QuesID: -13
Malar telangiectasia is a feature of?
1. Mitral stenosis
2. Systemic lupus erythematous
3. Floppy valve syndrome
4. Pseudoxanthoma elasticum
QuesID: -14
Blunted y descent is seen with?
1. Tetralogy of Fallot
2. Tricuspid stenosis
3. Tricuspid regurgitation
4. Tricuspid atresia

QuesID: -15
What happens to Austin flint murmur after exposure of vasodilators?
1. Accentuated
2. Softer
3. Longer duration
4. Shorter duration

QuesID: -16
Which of the following is not used for myocardial perfusion imaging?
1. Rubidium -82
2. 13N- ammonia
3. I-123 MIBG
4. Tetrofosmin- Tc 99
QuesID: -17
Which of these is best for myocardial viability and inflammation imaging?
1. PET
2. SPECT thallium 201
3. SPECT sestamibi Tc99
4. Coronary angiography

QuesID: -18
Agatson scoring is done for?
1. Coronary artery disease
2. End organ damage in hypertension
3. End organ damage in SLE
4. CHF with preserved

QuesID: -19
Triggered automaticity leads to development of which of the following
rhythm disorders?
1. Sinus bradycardia
2. Sinus tachycardia
3. Torsades de pointes
4. Ischemic ventricular fibrillation
QuesID: -20
Complete SA nodal block results in?
1. Progressive increase of PR interval followed by dropped beat
2. More than 3 different morphologies of P waves
3. Absent P waves
4. Unequal PP interval and RR interval

QuesID: -21
All of the following lead to AV block except?
1. Hypothyroidism
2. Ankylosing spondylitis
3. Lyme disease
4. Lev’s disease

QuesID: -22
Most common sustained tachycardia in healthy young women is?
1. AVNRT
2. AVRT
3. Atrial fibrillation
4. Pre-excitation syndrome
QuesID: -23
Increased AV nodal blockade leads to termination of tachycardia in all
except?
1. Atrial tachycardia
2. Atrial flutter
3. AVNRT
4. Orthodromic AV re-entry

QuesID: -24
Best for treatment of multi-focal atrial tachycardia is?
1. DC shock
2. Amiodarone
3. Verapamil
4. Beta blocker

QuesID: -25
ECG of a patient shows >3 consecutive premature ventricular contractions
(PVC) with a heart rate of less than 100 bpm. The diagnosis is?
1. Sustained VT
2. Non-sustained VT
3. Accelerated idioventricular rhythm
4. Ventricular flutter
QuesID: -26
A patient is admitted with a stab wound in the left anterior chest wall in the
parasternal line. Which cardiac chamber is most likely to be injured?
1. Right atrium
2. Right ventricle
3. Aorta
4. Left ventricle

QuesID: -27
A known case of WPW presents with palpitations and feeling uneasy.
Based on the clinical history and ECG findings, which of the following is the
most likely diagnosis?

1. Multifocal atrial tachycardia


2. Inappropriate sinus tachycar
3. Junctional tachycardia
4. Atrial fibrillation
QuesID: -28
A 67-year-old smoker is brought to the ER with severe abdominal pain and
hypotension. He has a history of hypertension, asthma and type-2 diabetes
mellitus. He currently takes Lisinopril and Metformin.
On examination, his heart rate is 100 bpm and his BP is 100/60mmHg. He
has normal first and second heart sounds and his chest is clear.
Which of the following risk factors is most likely to have contributed to the
development of his underlying condition?
1. Hypertension
2. Smoking
3. Diabetes
4. Family history of cardiovascular disease

QuesID: -29
Which of the following statements about coronary artery anatomy is
correct?
1. The left anterior descending artery runs around the atrioventricular
groove
2. Septal perforators originate from the right coronary artery
3. Majority of the patients have a left dominant coronary anatomy
4. Coronary artery dominance is defined by the artery which supplies
the inferior surface of the heart
QuesID: -30
A 68-year-old man presents with progressive shortness of breath. He had a
metallic heart valve replacement 13 years ago for severe aortic stenosis.
Which of the following clinical findings is most indicative of a failing aortic
valve replacement?
1. Clicking sound to S2
2. Early diastolic murmur heard at the left sternal edge
3. New onset atrial fibrillation
4. Ejection systolic murmur heard in the aortic region

QuesID: -31
A 60-year-old female is undergoing P.C.I after a coronary narrowing was
found on a previous diagnostic angiography. She is known to have
coronary vessels which are prone to spasm, therefore, the cardiologist
administers 2 puffs of sublingual GTN prior to the procedure and a further
400 mg of GTN during the procedure. During the procedure, the patient’s
systolic blood pressure suddenly drops to 70mmHg. At this point, the
patient is pale, clammy and complaining of light-headedness. The patient’s
pulse rate is noted to be 60bpm and regular.
What is the most appropriate management plan at this stage?
1. Administer a 150J defibrillator shock
2. Administer atropine
3. Administer metaraminol
4. Administer adrenaline
QuesID: -32
A 5-year-old boy is referred to a pediatric cardiologist after a check-up at
his GP revealed he had a murmur consistent with aortic regurgitation. What
is the MOST likely cause of this defect?
1. Atrial septal defect
2. Rheumatic fever
3. Cardiomyopathy
4. Ventricular septal defect

QuesID: -33
A patient has presented with palpitations. There no past history of any
similar episode(s) but he informs you that he consumed a large amount of
coffee at work. What is the most appropriate first-line treatment for this
patient?

1. Vagal maneuvers
2. Intravenous adenosine
3. Catheter ablation
4. Cardioversion
QuesID: -34
A 72-year old male is brought to A&E after an episode of loss of
consciousness at home. Upon examination, he is fully alert but appears
cold and clammy. His pulse is approximately 200 beats per minute and
blood pressure is 80/42 mmHg. The patient denies any chest pain, but
reports feeling extremely light headed. What does the ECG show?

1. Atrial fibrillation
2. Ventricular tachycardia
3. Asystole
4. Atrial flutter

QuesID: -35
A 25-year-old woman presents to the emergency with chest pain, shortness
of breath and hemoptysis for the past day. History and examination findings
raise the clinical suspicion of a pulmonary embolism (PE). Which of the
following individual factors would result in the highest Wells’s score?
1. Clinical signs and symptoms of DVT
2. Tachycardia and tachypnoea
3. Recent long-distance flight
4. Elevated D-Dimer
QuesID: -36
Which of the following can be used to calculate heart rate from an ECG?
1. (Number of large squares in one R-R interval)/300
2. 300/ (Number of large squares in one Q-T interval)
3. 300/ (Number of large squares in one R-R interval)
4. (Number of large squares in one Q-T interval) / 300

QuesID: -37
A 58-year-old man presents to you complaining of chest pain. The pain
occurs when he walks to the corner shop on his lunch break and
disappears within a couple of minutes when he stops to rest. He has never
had pain at rest or pain that did not spontaneously remit.
Clinical examination is normal except for mildly elevated blood pressure at
145/85. An ECG shows normal sinus rhythm. What is the most appropriate
investigation?
1. Fasting blood lipids
2. Urgent invasive coronary angiography
3. Repeat ECG in 1 week
4. CT coronary angiography
QuesID: -38
A frequent traveler presents with the following lesions. Comment on the
diagnosis?

1. Keratoderma blenorrhagicum
2. Psoriasis
3. Lichen Planus
4. Pompholyx

QuesID: -39
A mother brings her 5-year-old boy to see you as a GP. On examination,
he has red eyes, dry, cracked lips and a rash on his hands and feet. He
also has cervical lymphadenopathy.
What is the most important investigation to rule out a serious complication
of this condition?
1. Blood pressure
2. ECG
3. Echocardiogram
4. Blood tests for autoantibodies
QuesID: -40
Which of the following drugs can cause lupus due to low N-acetyl
transferase activity in the blood?
1. Propranolol
2. Hydralazine
3. Digoxin
4. Captopril

QuesID: -41
Which of the following is not true regarding radiocontrast media (RCM)
reaction?
1. Hypersensitivity reactions are independent of rate of infusion
2. Seizures and arrhythmias are noticed in RCM reaction
3. Physiologic reactions are related to the chemical properties of RCM
agents
4. Physiologic reactions are dependent on rate and date of infusion

QuesID: -42
A 20-year-old hypertension patient has been diagnosed as a case of
pheochromocytoma with positive biochemical evidence. What is the next
step in work up of this patient?
1. CT Abdomen
2. Perform urinary VMA levels
3. Perform adrenal vein sampling
4. MIBG scintigraphy
QuesID: -43
Uremia is not associated with hypertension in which of the following
situations?
1. Renal polyarteritis nodosa
2. Narrowing of the renal artery
3. Renal amyloidosis
4. Hyperplastic arteriolar nephrosclerosis

QuesID: -44
Renal biopsy in a patient of microscopic polyangiitis (MPA) will yield which
of the following findings?
1. Subepithelial humps on electron microscopy
2. Deposits showing apple-green birefringence on polarized light
3. Pauci-immune crescentic glomerulonephritis
4. Normal renal histology
QuesID: -45
A 60-year-old male with a past medical history of diabetes mellitus and
hypertension presented with fever, flank pain, and chills. He was treated for
pyelonephritis but the patient remained febrile after 5 days of antibiotics.
What is the next step in the management of this patient?
1. Change the current antibiotic therapy
2. Obtain a CT with contrast
3. Continue current antibiotic therapy
4. Repeat blood cultures
QuesID: -46
Which of the following is found in patients with cerebral salt wasting
syndrome?
1. Elevated serum cortisol
2. Hypernatremia
3. Hypovolemia
4. Low urine sodium excretion

QuesID: -47
Which is a contraindication to chemical cardioversion?
1. Heart rate faster than 150 beats per minute
2. Symptomatic palpitations
3. History of sinus pause after previous attempt to cardiovert with
adenosine
4. Hemodynamic instability

QuesID: -48
Why is chemical cardioversion considered for patients of recurrent
episodes of Atrial fibrillation?
1. It does not change the rhythm but improves the rate
2. It prevents the formation of a clot in the atrium
3. It is more effective than electrical cardioversion
4. It can be done electively without sedation
QuesID: -49
A 36-year-old male complains of a 2-day history of chest pain. The pain
has been continuous and does not change with activity. Vital signs show
normal blood pressure, mild tachycardia and tachypnea, and an oxygen
saturation of 90 % on room air. The cardiopulmonary exam is normal. An
ECG shows a rate of 106, left axis deviation, left ventricular hypertrophy,
PR depression, and T wave inversions in V2 to V5. Troponin and CPK-MB
are minimally elevated. What is the most likely diagnosis.
1. Pulmonary embolism
2. Costochondritis
3. Pericarditis
4. Unstable angina

QuesID: -50
Digoxin is contraindicated in?
1. Small VSD with heart failure
2. Viral myocarditis with heart failure
3. Atrial fibrillation with heart failure
4. Small ASD with heart failure
QuesID: -51
A 47-year-old female with a history of antiphospholipid antibody syndrome
who has been noncompliant with warfarin presents with a deep vein
thrombosis and dyspnea. She is hypotensive and tachypneic.
Computerized tomography of the chest shows a saddle embolus. She does
not respond to heparin and fluids. Echocardiogram shows right ventricular
hypokinesis. Select appropriate management.
1. Urgent referral for surgical embolectomy
2. Recombinant tissue plasminogen activator
3. Continue administration of fluids and heparin
4. Add lepirudin

QuesID: -52
Select the neurologic disorder not associated with antiphospholipid
antibodies.
1. Demyelinating disease
2. Hemorrhagic stroke
3. Headaches
4. Seizures

QuesID: -53
Which is not commonly caused by antiphospholipid syndrome?
1. Arterial and venous thromboembolic events
2. Frequent miscarriage
3. Hemolytic anemia
4. Hepatitis
QuesID: -54
Which of the following is false about Rheumatoid arthritis?
1. Often associated with hypertrophy of synovial membrane
2. Rheumatoid nodules are often painless
3. Ligament fibrosis is common early in the disease
4. The disorder cause permanent deformity

QuesID: -55
A female reports symmetrical small joint polyarthritis for 2 weeks. Labs
show rheumatoid factor levels at 1:320 (positive is 1:40) and anti-CCP at
58 units (40 to 59 units are considered strongly positive). An antinuclear
antibody test is negative. Labs also reveal positive cytomegalovirus and
paravovirus IgG, and negative parvovirus IgM. The ESR is 62 mm/hour.
What is the appropriate next step in the management of this patient?
1. Naproxen 500 mg twice a day and follow up in 1 month
2. Methotrexate 12.5 mg a week with liver function tests in 1 month
3. Anti-histone antibodies, anti-DS-DNA, and complement levels
4. Prednisone 60 mg a day and follow up in 2 weeks
QuesID: -56
A 77-year-old Asian woman mentions that she always develops skin
abscesses and oral ulcers following minor trauma. What is the most likely
diagnosis?
1. Pyoderma Gangrenosum
2. Behcet disease
3. Darrier’s disease
4. Bullous pemphigoid

QuesID: -57
Select the best initial treatment for a patient with Behcet syndrome and
ocular involvement
1. Topical glucocorticoids to the oral ulcers and conjunctiva
2. Thalidomide
3. Systemic glucocorticoids and azathioprine
4. Intralesional interferon A
QuesID: -58
Treatment of choice for late cardiovascular syphilis is?
1. Benzathine penicillin 2.4 million units in single dose i.m
2. Benzathine penicillin 7.2 million units in three divided doses i.m
3. Benzyl penicillin 12-24 million units for 21 days i.m
4. Tetracycline 2g daily
QuesID: -59
A healthy middle – aged man was arguing with his brother and got so
emotionally upset due to the arguments, that he suddenly developed chest
pain and collapsed. When brought to the hospital, he was declared dead.
What is the diagnosis?
1. Takotsubo cardiomyopathy
2. Dilated cardiomyopathy
3. Arrhythmogenic right ventricle dysplasia
4. Chronic ischemic cardiomyopathy

QuesID: -60
Most common complication of ICD is?
1. Delivery of unnecessary shocks
2. Device infection
3. Vascular occlusion
4. Cerebro-vascular accidents

QuesID: -61
In a patient with history of recurrent syncope, Holter recording shows QT
interval of 680msec. Which disease is suspected in this patient?
1. Long QT syndrome
2. Brugada syndrome
3. Arrhythmogenic RV dysplasia
4. Lev’s disease
QuesID: -62
Which of these is true about dabigatran?
1. Xa inhibitor
2. Excreted via liver
3. No reversal agent for bleeding
4. Inexpensive

QuesID: -63
25-year body builder was using anabolic steroids and started having puffy
feet for last 2 months. He died suddenly during a work out and post mortem
heart biopsy specimen report is shown below. Diagnosis is?

1. Dilated cardiomyopathy
2. Restrictive cardiomyopathy
3. Hypertrophic cardiomyopathy
4. Athlete's heart
QuesID: -64
Which is true regarding antiphospholipid antibody syndrome (APAS)?
1. Present in males more than females
2. Is autoimmune and associated with recurrent arterial and venous
thrombosis
3. Requires periodic anticoagulation
4. Not present in children

QuesID: -65
A 30 year old lady with crush injury was admitted to the casualty. Her
urobag shows 100ml red colour urine. ECG is shown below. All are
indicated for this patient except?

1. Hemodialysis
2. Furosemide drip
3. Calcium chloride
4. Magnesium sulfate
QuesID: -66
Atrial fibrillation may occur in all the following conditions, except?
1. Mitral stenosis
2. Hypothyroidism
3. Dilated cardiomyopathy
4. Mitral regurgitation

QuesID: -67
Feature of Torsade de pointes is?
1. Wide qRS complex
2. Short qRS complex
3. Prolonged QTc interval
4. Short QTc interval

QuesID: -68
Which of the following is a feature of hibernating myocardium?
1. Elevated cardiac troponins
2. Depressed ST segment
3. Elevated ST segment
4. Left bundle branch block
QuesID: -69
CXR was done for a 15 year old boy with low cardiac output. Which is
incorrect about the patient?

1. Reverse split S1
2. Straightening of left heart border
3. Carey Coombs murmur
4. Sea gull murmur

QuesID: -70
Which is incorrect about Rheumatoid arthritis?
1. Periarticular osteoporosis
2. Small joint involvement bilateral
3. Erosion of articular surface
4. Rheumatoid nodules is major criteria for diagnosis
QuesID: -71
Shrinking lung syndrome is seen in?
1. SLE
2. MCTD
3. Rheumatoid arthritis
4. Scleroderma

QuesID: -72
C.R.E.S.T syndrome is?
1. Generalised scleroderma
2. Limited scleroderma
3. Localised scleroderma
4. Systemic sclerosis

QuesID: -73
A 24 year old female presents with heliotrope rash and difficulty in climbing
stairs. All investigations are useful except?
1. Anti- Jo- 1 antibody
2. CPK MM
3. C3 levels
4. CT abdomen
QuesID: -74
Elevated JVP is seen in all except?
1. PAH
2. TOF
3. Mitral stenosis
4. Aortic stenosis

QuesID: -75
Myocardial stunning pattern which is not matching with ECG findings is
seen in?
1. Brugada Syndrome
2. Restrictive Cardiomyopathy
3. Pericardial Shudder
4. Tako-Tsubo cardiomyopathy

QuesID: -76
Hypertension with hypokalemic alkalosis is seen in?
1. Liddle syndrome
2. Gitelman syndrome
3. Bartter syndrome
4. Fanconi syndrome
QuesID: -77
Pseudo- P- pulmonale is seen in?
1. Hypokalemia
2. Hyperkalemia
3. Hypomagnesemia
4. Hypercalcemia

QuesID: -78
A 35-year-old male with history of immobilization for fracture femur
presents with sudden onset chest pain and haemoptysis. Urgent CT chest
shows?

1. Aortic dissection
2. Acute Cor-pulmonale
3. Fat embolism
4. Acute respiratory distress syndrome
QuesID: -79
Which of the following is called “flipping effect”?
1. LDH 1 >LDH 2
2. LDH 2 > LDH 1
3. LDH 2 > LDH 3
4. LDH 3 > LDH 2

QuesID: -80
Tadalafil is contraindicated in?
1. Pulmonary artery hypertension
2. Chronic stable angina (acute episode)
3. Active infections
4. Hepatic dysfunction

QuesID: -81
Which of the following is associated with A-V Block?
1. Hypothyroidism
2. Hyperthyroidism
3. Cushing disease
4. Pheochromocytoma
QuesID: -82
Mobitz II heart block is seen with all except?
1. Hypothyroidism
2. Coronary artery disease
3. Sarcoidosis
4. Cushing syndrome

QuesID: -83
Intracardiac defibrillator is useful in management of?
1. Arrhythmogenic RV dysplasia
2. Brugada syndrome
3. LV aneurysm (Post MI)
4. All of the above

QuesID: -84
Neprilysin inhibitor is?
1. Conivaptan
2. Nesiritide
3. Sacubitril
4. Aliskiren
QuesID: -85
Best management of a hemodynamically stable patient with ECG showing
Broad QRS complex with antidromic tachycardia is?
1. Oral Verapamil
2. Oral Beta-blocker
3. Cardioversion
4. Intravenous Procainamide

QuesID: -86
Which of the following is not done in W.P.W?
1. Treadmill test
2. Electrophysiological studies
3. Oral beta blocker
4. Procainamide

QuesID: -87
Rescue P.C.I is done for which of the following?
1. Persistent chest pain with ST elevation > 60 min after thrombolysis
2. Persistent chest pain with ST elevation > 30 min after thrombolysis
3. Persistent Chest Persistent Chest pain with ST elevation >90 min
after thrombolysis
4. Pain with ST elevation for >120minutes after thrombolysis
QuesID: -88
What is the impact on fetus in case of use of Indomethacin in utero in third
trimester?
1. Patent ductus arteriosus
2. Early closure of ductus arteriosus
3. Ventricular septal defect
4. Atrial septal defect

QuesID: -89
Waist to hip ratio that increases risk of heart disease is?
1. > 0.80 in male
2. > 0.80 in female
3. > 0.85 in male
4. > 0.85 in female

QuesID: -90
Which of the following is not correct about Defibrillation?
1. Easy for untrained person
2. Decrease in success rate with delay in initiation
3. Improves prognosis
4. 1-minute gap between 2 shocks
QuesID: -91
The following patient has presented after chest trauma. On examination
crepitus is felt. The clinical diagnosis is?

1. Subcutaneous Emphysema
2. Gas gangrene
3. Acute tubular necrosis
4. Cardiac Tamponade

QuesID: -92
Comment on the diagnosis of the image shown.

1. Pneumoperitoneum
2. Pneumothorax
3. Eventration of diaphragm
4. Pneumomediastinum
QuesID: -93
Duke’s Score is used for evaluation of?
1. Infective endarteritis
2. Severity of aortic stenosis
3. Chronic Stable Angina
4. Infective endocarditis

QuesID: -94
Which of the following shall be seen with use of a small size BP cuff?
1. False elevation of BP
2. Falsely low value of BP
3. Cancels the effect of calcified arteries
4. Increases trans-arm impedance

QuesID: -95
Elderly male patient has presented with recurrent falls. Which of the
following medicines is responsible?
1. Prazocin
2. Metformin
3. Acarbose
4. Thiazides
QuesID: -96
On putting an Internal jugular vein catheter, a patient has developed
sudden onset severe respiratory distress. Clinical diagnosis is?
1. Pneumothorax
2. Sepsis
3. ARDS
4. Infective Endarteritis

QuesID: -97
Besides ECG, what other studies are required to study diagnosis Brugada
syndrome?
1. Stress test
2. Holter monitor
3. Angiogram
4. Echocardiography

QuesID: -98
What type of mutation is most commonly associated with Brugada
syndrome?
1. Gain of function mutation
2. Loss of function mutation
3. Antimorphic mutation
4. Lethal mutation
QuesID: -99
A 65-year-old man is about to undergo an elective hip replacement. A pre-
operative ECG is taken (see below). Based upon the ECG, which of the
following jugular venous pressure (JVP) abnormalities is most likely to be
present?

1. Absent A waves
2. Cannon A waves
3. Giant V waves
4. Rise in JVP during inspiration

QuesID: -100
A 68 -year-old male, known case of HTN is brought unconscious. Upon
arrival, the patient is found to have a Blood Pressure of 245/142. You
decide to assess the patient for end-organ damage. Which of the following
investigations would be least useful?
1. Serum Creatinine
2. CT pulmonary angiogram
3. 12-lead ECG
4. Fundoscopy
QuesID: -101
Where is the cardiac plexus located?
1. Posterior to the bifurcation of the trachea
2. Anterior to the bifurcation of the trachea
3. Anterior to the left heart border
4. Posterior to the apex of the heart

QuesID: -102
A 26-year-old man attends an occupational heath assessment in
preparation for new employment. A 12 leads ECG is recorded as shown.
He is feeling well and is experiencing no symptoms.
He has no significant past medical history. Based on the ECG findings what
is the most likely diagnosis?

1. Right Bundle Branch Block


2. Wolff-Parkinson White Syndrome
3. Brugada Syndrome
4. Normal Variant
QuesID: -103
A 70-year-old man with a past medical history of ischemic heart disease
and hypertension presents to his GP with recurrent episodes of syncope
over the past 2 weeks.
Based upon this ECG, which of the following clinical findings is most likely
to be present?

1. Absent JVP
2. Canon A waves
3. Absent hepato-jugular reflux
4. Giant V waves

QuesID: -104
A 56-year-old female presents with severe chest pain and breathlessness
after coming back from cremation grounds due to death of her husband.
ECG is performed which shows ST-segment elevation in the precordial
leads, most noticeably in V2-V3. As a result, cardiac enzymes (Troponin T)
and BNP are requested which are mildly elevated. The patient
subsequently undergoes invasive coronary angiography which shows
completely normal coronary arteries. A transthoracic echocardiogram
demonstrates apical akinesis and a left ventricular ejection fraction of 40% .
What is the most likely diagnosis?
1. Anxiety
2. Septal myocardial infarction
3. Coronary artery spasm
4. Takotsubo cardiomyopathy

QuesID: -105
You are asked to review the ECG below, for a 72-year-old male who has
presented to ER with falls. Based on your ECG interpretation, what is the
likely diagnosis?

1. Hyperkalaemia
2. Second degree – Mobitz Type 2 – heart block
3. First degree heart block
4. Second degree – Mobitz Type 1 – heart block

QuesID: -106
A 30-year-old female presents with a 3-month history of 10kg weight loss
despite an increased appetite. She also complains of anxiety, diarrhea and
amenorrhea. Examination reveals a fine tremor, brisk reflexes and a
systolic murmur heard throughout the precordium. A urine pregnancy
screen was negative. What is the most likely finding on examining the
pulse?
1. Pulsus paradoxus
2. Collapsing pulse
3. Irregularly irregular pulse
4. Pulsus alternans
QuesID: -107
What does the ECG show?

1. Mobitz type 1 heart block


2. First degree heart block
3. Complete heart block
4. Mobitz type II heart block

QuesID: -108
A 25-year-old male who is an intravenous drug user presents with a 3-week
history of worsening lethargy and confusion. Examination findings show a
Pulse of 130bpm and Blood pressure: 120/80
CVS examination reveals a raised JVP with large v-waves and a loud
systolic murmur. A palpable, pulsatile liver is felt when hepato-jugular reflex
is attempted. What type of murmur is most fitting with this patient’s
presentation?
1. Functional murmur
2. Tricuspid stenosis
3. Pulmonary stenosis
4. Tricuspid regurgitation
QuesID: -109
Mrs Sharma , a retired government officer has recently been diagnosed
with non-valvular atrial fibrillation. When considering the need for starting
anti-coagulation therapy which scoring system can be used to calculate her
percentage 1 year risk of developing a major bleed?
1. HEART
2. HASBLED
3. CHADS2 –Vasc
4. NIHS

QuesID: -110
A 78 year old male is admitted with chest pain. His ECG shows widespread
ST depression in all leads except aVR which shows ST elevation. Whilst in
ER, he passes melaena and, on further questioning, admits to a history of
altered bowel habit, weight loss and dark stools for 2-3 months. On
examination, he is pale. His pulse rate is 115bpm and his blood pressure is
80/50mmHg. Blood tests demonstrate a hemoglobin of 36g/l and a high-
sensitivity troponin of 140ng/l. what is the best initial management for this
guy?
1. Aggressive treatment with dual antiplatelets
2. Packed RBC transfusion
3. Nitrates to relieve ischemia
4. Beta blocker to slow heart rate and reduce ischaemia
5. Urgent coronary angiography +/- angioplasty

You might also like